Login

Welcome, Guest. Please login or register.

March 29, 2024, 08:40:16 am

Author Topic: VCE Physics Question Thread!  (Read 603375 times)  Share 

0 Members and 3 Guests are viewing this topic.

BasicAcid

  • Victorian
  • Forum Obsessive
  • ***
  • Posts: 207
  • Respect: +135
Re: VCE Physics Question Thread!
« Reply #300 on: October 30, 2013, 10:36:52 pm »
0
Lenz's law, the current produced opposes that of the change in flux

lzxnl

  • Victorian
  • ATAR Notes Legend
  • *******
  • Posts: 3432
  • Respect: +215
Re: VCE Physics Question Thread!
« Reply #301 on: October 30, 2013, 10:37:53 pm »
0
North pole moves to left=>induced magnetic field points to the the right=>use right hand rule, current is from q to p in the EXTERNAL circuit, but p to q through the ammeter.
2012
Mathematical Methods (50) Chinese SL (45~52)

2013
English Language (50) Chemistry (50) Specialist Mathematics (49~54.9) Physics (49) UMEP Physics (96%) ATAR 99.95

2014-2016: University of Melbourne, Bachelor of Science, Diploma in Mathematical Sciences (Applied Maths)

2017-2018: Master of Science (Applied Mathematics)

2019-2024: PhD, MIT (Applied Mathematics)

Accepting students for VCE tutoring in Maths Methods, Specialist Maths and Physics! (and university maths/physics too) PM for more details

Jaswinder

  • Victorian
  • Trendsetter
  • **
  • Posts: 152
  • Respect: 0
  • School Grad Year: 2014
Re: VCE Physics Question Thread!
« Reply #302 on: October 30, 2013, 10:39:16 pm »
0
North pole moves to left=>induced magnetic field points to the the right=>use right hand rule, current is from q to p in the EXTERNAL circuit, but p to q through the ammeter.

whats the external circuit and how is it different to this one  :-\ :-[

lzxnl

  • Victorian
  • ATAR Notes Legend
  • *******
  • Posts: 3432
  • Respect: +215
Re: VCE Physics Question Thread!
« Reply #303 on: October 30, 2013, 10:51:54 pm »
0
The external circuit is the solenoid that the ammeter is connected to.
2012
Mathematical Methods (50) Chinese SL (45~52)

2013
English Language (50) Chemistry (50) Specialist Mathematics (49~54.9) Physics (49) UMEP Physics (96%) ATAR 99.95

2014-2016: University of Melbourne, Bachelor of Science, Diploma in Mathematical Sciences (Applied Maths)

2017-2018: Master of Science (Applied Mathematics)

2019-2024: PhD, MIT (Applied Mathematics)

Accepting students for VCE tutoring in Maths Methods, Specialist Maths and Physics! (and university maths/physics too) PM for more details

Robert123

  • Victorian
  • Forum Obsessive
  • ***
  • Posts: 201
  • Respect: +5
  • School: Kyabram P-12 College
Re: VCE Physics Question Thread!
« Reply #304 on: October 31, 2013, 02:04:43 pm »
0
Having some difficulties with lenzs law.
A loop is sitting in a magnetic field that is going out of the page, it is quickly pulled to the left. Which direction will the current flow (clockwise or anticlockwise)?
Looking at the left hand side of the loop, you can use the right hand slap rule to determine the direction of the current( fingers going out of the page, palm to the right to oppose the force to  the left, current will be clockwise). However, if you do the same analysis on the other side, the thumb would be pointing the same way so current will be anticlockwise. Which one is correct and why?

Stevensmay

  • Guest
Re: VCE Physics Question Thread!
« Reply #305 on: October 31, 2013, 09:25:47 pm »
0
Having some difficulties with lenzs law.
A loop is sitting in a magnetic field that is going out of the page, it is quickly pulled to the left. Which direction will the current flow (clockwise or anticlockwise)?
Looking at the left hand side of the loop, you can use the right hand slap rule to determine the direction of the current( fingers going out of the page, palm to the right to oppose the force to  the left, current will be clockwise). However, if you do the same analysis on the other side, the thumb would be pointing the same way so current will be anticlockwise. Which one is correct and why?

As the loop is moved out of the field, the amount of flux it encloses decreases. Lenz law then tells us that there will be an induced current that will oppose this decrease in flux by increasing it.

To increase flux we want to generate a B-field that is in the same direction as the original B-field, out of the page. Using our right hand grip rule, we can see that to generate a B-field going out of the page within the loop, the current must be flowing anti clockwise.

Can you please check this with your answer? I'm not totally happy with my reasoning.


Robert123

  • Victorian
  • Forum Obsessive
  • ***
  • Posts: 201
  • Respect: +5
  • School: Kyabram P-12 College
Re: VCE Physics Question Thread!
« Reply #306 on: November 01, 2013, 07:43:58 am »
0
Yes The answer agree with that. I just want to know how to do using the RH slap rule as it could provide a quick answer check.

Also, with that reasoning, does it matter if the coil is move to the LEFT or to the RIGHT?

lolipopper

  • Victorian
  • Forum Obsessive
  • ***
  • Posts: 317
  • I'm making aaaalll kaaindzzz of gaains
  • Respect: -4
  • School: Lalor North Secondary College
  • School Grad Year: 2013
Re: VCE Physics Question Thread!
« Reply #307 on: November 02, 2013, 12:17:38 am »
0
are questions 9-11 from 2007 VCAA U4 AOS2 still in course?

also what is the range of wavelength-gap ratio that we should assume that allows diffraction, because in the same exam Q5 i got the ratio to be 0.667 and thought this was much lower than 1, hence no diffraction.

http://www.vcaa.vic.edu.au/Documents/exams/physics/2007physics2.pdf
2014: Monash University, Law

SocialRhubarb

  • Victorian
  • Forum Obsessive
  • ***
  • Posts: 313
  • Respect: +34
  • School Grad Year: 2013
Re: VCE Physics Question Thread!
« Reply #308 on: November 02, 2013, 01:48:53 am »
+1
Question 9 would be an application of knowledge of electron energy levels in atoms, and its relation to light. In the course, but clearly the difficult here is recognising what the question needs.

I think incandescent lightbulbs are off the course, but you should understand the principles behind a vapour lamp, and how they work.

Quote from the study design: explain the production of atomic absorption and emission spectra, including those from metal
vapour lamps.

We were told that anything above 0.1 will give you noticeable diffraction. A ratio of 0.667 would give you about 45 degrees of diffraction, definitely enough to be noticed.
Fight me.

~T

  • Victorian
  • Trendsetter
  • **
  • Posts: 197
  • Respect: +4
  • School: St Patrick's College
  • School Grad Year: 2013
Re: VCE Physics Question Thread!
« Reply #309 on: November 02, 2013, 10:40:21 am »
0
On that note, what happens to diffraction when the ratio becomes larger than one? Ie. Wavelength > Slit size

Does the diffraction become even more significant, or does it gradually decrease again, or does it stop entirely? (perhaps because the wave is "too large"???)
ATAR: 99.95
Specialist 50 | Methods 50 | Physics 50 | Further 49 | Literature 48 | Music Style/Composition 41

2014 - 2016: Bachelor of Science (Chancellor's Scholars' Program) at The University of Melbourne

I will be tutoring in Melbourne this year. Methods, Specialist, and Physics. PM me if you are interested :)

lzxnl

  • Victorian
  • ATAR Notes Legend
  • *******
  • Posts: 3432
  • Respect: +215
Re: VCE Physics Question Thread!
« Reply #310 on: November 02, 2013, 10:46:40 am »
+1
For diffraction through a single slit, the angular positions of the intensity minima are given by
where m is a constant (not zero), d is the slit width and theta is the angle the line from the slit to the minima makes with a line parallel to the slit opening.
As you can see, if the wavelength approaches the slit width, the first angle approaches ninety degrees, which means the first intensity minima spreads out completely. If the wavelength exceeds the slit width, then you will not have any minima and the wave will spread completely.


*begins rant about VCE physics
2012
Mathematical Methods (50) Chinese SL (45~52)

2013
English Language (50) Chemistry (50) Specialist Mathematics (49~54.9) Physics (49) UMEP Physics (96%) ATAR 99.95

2014-2016: University of Melbourne, Bachelor of Science, Diploma in Mathematical Sciences (Applied Maths)

2017-2018: Master of Science (Applied Mathematics)

2019-2024: PhD, MIT (Applied Mathematics)

Accepting students for VCE tutoring in Maths Methods, Specialist Maths and Physics! (and university maths/physics too) PM for more details

sin0001

  • Victorian
  • Forum Obsessive
  • ***
  • Posts: 487
  • Respect: +1
  • School Grad Year: 2013
Re: VCE Physics Question Thread!
« Reply #311 on: November 02, 2013, 10:57:39 am »
0
For those generic questions asking how Young's double-slit experiment supports the wave model of light, I talk about how a fringe pattern is observed and hence light is shown to interfere, which is a wave phenomenon and thus supports this model. But is it also correct to mention how diffraction is observed, because theoretically electrons/other particles also have De Broglie wavelengths and can diffract through the slits?
Don't see many company solutions, and even VCAA, mentioning diffraction for these questions.
ATAR: 99.00
Monash Commerce Scholars

SocialRhubarb

  • Victorian
  • Forum Obsessive
  • ***
  • Posts: 313
  • Respect: +34
  • School Grad Year: 2013
Re: VCE Physics Question Thread!
« Reply #312 on: November 02, 2013, 12:01:03 pm »
0
I guess?

The thing is that it's a double slit diffraction experiment that you're being asked about. They've given you two slits, why only talk about single slit diffraction? It's clear that the question is directing you to talk about interference patterns, and if it asks you specifically what Young's double-slit experiment shows, the most notable thing about it is the interference pattern it shows. If you wanted an experiment to show simple diffraction of light, you probably wouldn't use Young's double-slit experiment.

I'm not sure what point you're trying to make about electrons and de Broglie wavelengths. The fact that electrons can diffract, and also interfere, speaks about the wave nature of matter, not the nature of light.
Fight me.

sin0001

  • Victorian
  • Forum Obsessive
  • ***
  • Posts: 487
  • Respect: +1
  • School Grad Year: 2013
Re: VCE Physics Question Thread!
« Reply #313 on: November 02, 2013, 12:06:57 pm »
0
I guess?

The thing is that it's a double slit diffraction experiment that you're being asked about. They've given you two slits, why only talk about single slit diffraction? It's clear that the question is directing you to talk about interference patterns, and if it asks you specifically what Young's double-slit experiment shows, the most notable thing about it is the interference pattern it shows. If you wanted an experiment to show simple diffraction of light, you probably wouldn't use Young's double-slit experiment.

I'm not sure what point you're trying to make about electrons and de Broglie wavelengths. The fact that electrons can diffract, and also interfere, speaks about the wave nature of matter, not the nature of light.
Ah okay, what I was trying to ask was whether *only* mentioning interference would suffice, or is it better to also talk about diffraction.
ATAR: 99.00
Monash Commerce Scholars

ECheong

  • Victorian
  • Forum Regular
  • **
  • Posts: 65
  • Respect: +1
  • School: Nossal High School
  • School Grad Year: 2013
Re: VCE Physics Question Thread!
« Reply #314 on: November 02, 2013, 12:26:59 pm »
0
I guess if you wanted to be very safe you could say that the interference patterns can only exist if diffraction happened, i.e. creating path differences between the light that passes through Slit A and from Slit B and therefore, the two waves from each respective slit interfering?

On the whole though, I'd personally focus on constructive and destructive interference as that's typically what they want from an answer (judging by examiners reports). As long as its good physics you can't be pinged :)
BBiomedSci/LLB (Monash)